What is Magnetic force: Definition and 413 Discussions

In physics (specifically in electromagnetism) the Lorentz force (or electromagnetic force) is the combination of electric and magnetic force on a point charge due to electromagnetic fields. A particle of charge q moving with a velocity v in an electric field E and a magnetic field B experiences a force of





F

=
q


E

+
q


v

×

B



{\displaystyle \mathbf {F} =q\,\mathbf {E} +q\,\mathbf {v} \times \mathbf {B} }
(in SI units). It says that the electromagnetic force on a charge q is a combination of a force in the direction of the electric field E proportional to the magnitude of the field and the quantity of charge, and a force at right angles to the magnetic field B and the velocity v of the charge, proportional to the magnitude of the field, the charge, and the velocity. Variations on this basic formula describe the magnetic force on a current-carrying wire (sometimes called Laplace force), the electromotive force in a wire loop moving through a magnetic field (an aspect of Faraday's law of induction), and the force on a moving charged particle.
Historians suggest that the law is implicit in a paper by James Clerk Maxwell, published in 1865. Hendrik Lorentz arrived at a complete derivation in 1895, identifying the contribution of the electric force a few years after Oliver Heaviside correctly identified the contribution of the magnetic force.

View More On Wikipedia.org
  1. C

    Force on rectangular loop by a current in a long straight wire

    Hi, I am struggling to get the right answer for this question. My first thought was that I should consider to what direction does each segment of wire have a force towards. I found the direction to be in the following (see red arrows): My past attempt was: Floop = IlooplloopBwire Since Bwire...
  2. R

    Net magnetic force on circuit segment due to 2 parallel wires

    This answer is incorrect when I try submitting it.
  3. Z

    Understanding work in translation and rotation of a magnetic dipole

    these notes on magnetic field from MIT OCW's 8.02 course. There is the following snippet on page 10 At this point it is still not clear to me exactly what a magnetic dipole is. I've done all the calculations for a current-carrying rectangular loop, so I will assume that a magnetic dipole is...
  4. walkeraj

    B A Magnetic Misconception on Divergence 0/Closed Field Lines?

    Question: Can we ultimately atttribute no work or net zero work done by a magnetic force to the closed magnetic field lines that results in Divergence zero of a magnetic field? That is, is it a misconception to say that closed magnetic field lines imply magnetic force will always result in no...
  5. A

    Electromagnetism: Force on a parabolic wire in uniform magnetic field

    I know the easier method/trick to solve this which doesn't require integration. Since parabola is symmetric about x-axis and direction of current flow is opposite, vertical components of force are cancelled and a net effective length of AB may be considered then ##F=2(4)(L_{AB})=32\hat i## I...
  6. S

    I Calculation for proportional mechanical solenoid Force

    I'm looking for calculation for proportional solenoid force (constant force for some working stroke) using magnetic reluctance based on equivalent magnetic circuit by networking method. I was reading a book Elektromangnate by Dr Kallenbach which discussed such method but for switching solenoid...
  7. ktmsud

    I Hall effect -- is it always applicable?

    A current carrying conductor experiences magnetic force in a magnetic field. F=BILsinθ Where, B = Magnetic flux density I = Current L = Length of conductor and θ = Angle between magnetic field and current This force is due to free electrons moving in a...
  8. Kupa140

    Analyzing the Force of Two Disk Magnets on a Metal Plate

    The situation is as follows. We have two disk magnets. One is fixed on the ground, table, or surface and has the north pole facing up. Then we have a metal plate fixed on the vertical axis rod or something similar such that it can't move up or down, but only rotate horizontally with as less...
  9. K

    How can I troubleshoot and verify my calculations for magnetic force?

    I am getting a wrong answer for part b, however part a and c both are correct : a. 0 N b = -2.91E-3 N (WRONG) c. 1.4E-4 N
  10. D

    I Question on Hall Effect and magnetic force

    so with a Hall Voltage, you have positive current traveling upwards in a wire in the +y-direction and a magnetic field into the screen in the -z-direction. the right hand rule has positive charge deflecting to the left. now if you look at the drift velocity of electrons moving downward in the...
  11. N

    I Exploring Magnetic Force: Magnet vs. Wires

    I understand the iron fillings become little magnets all pointing in the same north south direction similar to the spin aligned electrons in the permanent magnet. Similarly, a compass near a wire traces out the magnetic field lines ie North/South. My question is how do I reconcile the fact that...
  12. L

    Conducting rod in equilibrium due to magnetic force

    I am having problems understanding point (b) so I would like to know if my reasoning in that part is correct and/or how to think about that part because I don't see how to justify the assumption ##v_y=0\ m/s##. Thanks. I set up the ##xyz## coordinates system in the usual way with ##xy## in the...
  13. B

    Velocity of a relativistic particle in a uniform magnetic field

    d(ɣmv)/dt = qvB (dɣ/dt)mv + ɣm(dv/dt) = qvB Substituting gamma in and using the chain rule, it ends up simplifying to the following: ɣ^3*m(dv/dt) = qvB Now, I am confused on how to solve for v.
  14. J

    Earth's magnetic force on a proton

    Plug in the elementary charge for q, 1000 m/s for v, 50 microtesla for B and 90 degrees for theta and I get about 8e-21 Netwons. But apparently this is wrong, anyone know why? Thank you.
  15. rayjbryant

    I Quantifying the magnetic force on a magnet moving through a coil?

    So I'm familiar with the magnet falling through a copper tube demonstration that shows the induced magnetic fields slowing the magnet down. I know that this experiment is also possible with a conducting coil as long as the coil forms a closed circuit. I'm trying to find a way to calculate the...
  16. Elementrist

    B Magnetic force driving a small iron ball

    How can I calculate the force from magnetic field of a solenoid, grabbing a small iron ball? I want to use the good old simple F=ma formula in order to calculate the ball's acceleration. But can't find a formula to somehow convert the known field quality (in unit Tesla) to Force (in unit...
  17. S

    Magnetic energy density, and pressure due to magnetic force

    Hi, The problem I am working on requires me to work out the the pressure on the outer conductor of a coaxial cable due to the current on the inner one. This cable carries a dc current of 5000 Amps on the inner wire of radius 2 cm. The outer cylindrical wire of radius 5cm carries the return...
  18. Viona

    Why Doesn't Electric Force Cause Wires to Move?

    Hello, In the section of Magnetic Force on a Current- Carrying Conductor in the book of College Physics by Serway, it is written that the Current- Carrying Conductor in a magnetic field deflects because the magnetic force on the electrons transfers to the bulk of the wire due to the collisions...
  19. archaic

    Exploring the Magnetic Force and Torque on a Current-Carrying Wire

    a) On QP, the force is zero. On PR, the force is directed inside the plane, and its magnitude is ##0.8IB##. On QR, the force is direction out of the plane, and its magnitude is ##1IB\sin(\arctan(0.8/0.6))=0.8IB##. b) ##0.8IB-0.8IB=0##. d) It will rotate outside of the page. c) I know that the...
  20. archaic

    Cylinder rolling due to magnetic force

    Hello! The magnetic force is to the right. ##I_c## is the moment of inertia of the cylinder. For the net force on the centre of mass, I have the frictional and magnetic forces ##F=F_B-f##. I know that ##F_B## is ##IdB##. I also know that ##rf=I_c\alpha=I_c\frac ar##, so that...
  21. P

    Finding the Magnetic force on the coil

    Surely a tough one, I am doing it from the basics. This is the diagram i tried to draw showing the Force and current I The Length L is the tangent to the circle. The Force F is pointing upwards at ##90 Deg## to the ##\vec B## and also perpendicular to ##\vec L##. I am considering a small...
  22. tanaygupta2000

    Angle of deviation from a magnetic field

    The beam of protons are directed towards the axis of the cylinder, perpendicular to the direction of the field. While traveling through the cross-section of the cylinder, the proton beam experiences a magnetic force, which tends to move the beam in a circular orbit of the radius given by: r =...
  23. jaumzaum

    How Does Magnetic Force Affect a Falling Coil's Velocity and Kinetic Energy?

    Consider a coil perpendicular to the ground falling with gravity. Under it, there is a magnetic field also perpendicular to the coil. When the coil starts penetrating the magnetic field there will be an induced current and therefore a magnetic force upwards. This magnetic force will reduce...
  24. jisbon

    Dealing with magnetic force into the page?

    Since the magnetic field is pointing down, I can derive that the magnetic force is into the page. With this, I can't really imagine how the particle can move in a circular motion as I cannot visualise how the centripetal force will look like when the force is into the page (instead of usual...
  25. D

    How to find the magnetic field and magnetic force due to a solenoid loop

    I'm not so sure how to begin with this problem. I was thinking of usign superposition. I think that the field on the conductor due to the parallel segments of the coil is zero, since Ampere's Law tells us that the field outside the solenoid is zero, right? For the perpendicular segments, I used...
  26. K

    I Equation for levitation forces between the magnets and superconductor

    How can I measure the levitation forces between the magnets and a type-II superconductor (YBCO superconductor)?
  27. Haorong Wu

    Why -e(v/c)H is the magnetic force?

    In a paper I am reading, it reads, "For these orbits the electric force ##-e E_r## almost balances the magnetic force ##-e \left ( v/c \right ) H_0##." where ##-e## is the charge of the electrons, ##v## is the speed of the electrons, ##H_0## is a homogeneous magnetic field, and ##c## is not...
  28. fight_club_alum

    Exploring Magnetic Forces on Current: A Visual Guide

    This is how I visualize the problem (of course I am drawing this as if it is in the z-y axis); I don't know what will be the next step. Anyone please help me. Thank you
  29. fight_club_alum

    Calculating Magnetic Force with Vectors

    8000 0 0 5 -4 3 F = (5 x 10^-6) * up (sorry can't write a materix here) F = (5 x10^-6) {0i - 24000 j - 32000 k) F = -0.12 j - 0.16 k Mag of F = 0.2 (I feel that there is something wrong in the question; I don't know)
  30. N

    Magnetic force on a wire due to a loop

    I just need to confirm my intuition that the magnetic force on the wire carrying I2 is 0. Basis for my intuition: * Right above the center of the loop carrying I1, the magnetic field lines are in exactly the same direction as the piece of wire carrying I2, so [a] x [B1] = 0. (photo from...
  31. Y

    Magnetic force -- does it fall off to zero far away from the source?

    Let's say we have a wire and a circuit is closed the magnetic field start to spread with speed of light c. Then we would have a place " out of range " at which the magnetic field doesn't exist.What is the equation that connects magnetic force with distance with its prorogation of speed of light...
  32. B

    How would you find the magnetic force on a piece of metal by an electromagnet?

    Does the metal being attracted make a difference? How would you find the force on the metal knowing the field strength there? How would you even calculate the field strength of an electromagnet with a core? In my case, it is one of those junkyard electromagnets. thanks
  33. JD_PM

    Arguing about the magnetic force vector

    I am writing about the nature of force in classical mechanics and what does really imply, in terms of change in motion. I am using as an example a circuit, on which we exert a force. I am trying to justify the following scheme (concretely, ##f_{mag}##): The thing is that I am wondering how...
  34. J

    Loop falling in a magnetic field

    (a) Let's say the loop has fallen ##y## from its initial position. Then the magnetic flux is ##B_{0}w(h-y)## and the induced voltage is ##\mathcal{E}=B_{0}wdy/dt##. Since this voltage is positive, the current flows clockwise. (b) ##I=\frac{\mathcal{E}}{R}=\frac{B_{0}wv}{R}## (c) The force on...
  35. MedEx

    Find the Velocity given Magnetic field and Magnetic Force

    Homework Statement A proton moves through a uniform magnetic field given by B=(13.7i−22.9j+23.5k) mT. at time t1 v=(vxi +vyj+1.68k)km/s and the magnetic force is given by F= (4.22e-17i + 2.52e-17j). What are vx and vy Homework Equations F=q(v x B) The Attempt at a Solution First I converted...
  36. Zack K

    I Do two perpendicular wires exert a force on each other?

    I have a question for my E&M assignment (I'm not putting it in the homework thread since I just need more of a concept check) that involves a straight wire with current through it. If you have a wire with a current through it, and you have a rectangular wire also with current, to the right of...
  37. H

    The work of magnetic force on the wire

    When the two current carrying wires (with identical current directions) attract each other, it seems that the magnetic field of wires do work while we know that magnetic force is perpendicular to the velocity of charge. I would like to know whether the the two wires get closer or not? What about...
  38. A

    Is the Lorentz magnetic force verified?

    Can someone provide me with some original experiments conducted which verified the Lorentz magnetic force : F = q(v X B). Google does not give me any links except to explanations of the Lorentz force law.
  39. S

    Magnetic force between an unknown and known permanent magnet

    Dear helpers, I working on an projekt for the University and for that I have to calculate forces between different magnets. The question that I have is: "Is there a way to calculate the force between two magnets if one is completely known and the other unkown. But you know the B-field of the...
  40. YMMMA

    Magnetic force direction on a positive and negative charges

    Homework Statement The question askes about the path of the electron. I need to know if I am doing it correctly whether it was asked for a proton or an electron. Homework Equations F=QV⊥Bsinθ The Attempt at a Solution I answered it C. Because it is an electron, I used my left hand. My thumb...
  41. F

    How scientists ensured this fundamental property of magnets?

    Firstly, please note that I am talking about the period BEFORE electricity and magnetism were unified. So I am NOT seeking for answers based on Ampere atomic current model of magnets. I have read the following statement about the property of magnets at two different places. One from here: and...
  42. CMW328i

    Engineering Help with magnetic field forces in a motor

    Hi all, Not a question about completing homework here, but I'm a teacher looking to create a realistic engineering question for an assignment. I have an engineering scenario I've set for the assignment which is a junior engineer working for a marine engineering company so all of the questions...
  43. V

    Mass needed to balance the magnetic force on upper rod

    Homework Statement Two straight rods 60 cm long and 2.0 mm apart in a current balance carry currents of 18 A each in opposite directions. What mass must be placed on the upper rod to balance the magnetic force of repulsion? ## \mu_0 = 4 \pi * 10^-7 ~\frac {T * M} {A}## ## g = 9.81 ~m/s ##...
  44. S

    How to find the magnetic force on a current-carrying tube in a neon sign?

    1. The problem statement, all variables, and given/known data Assume the Earth’s magnetic field is 52.0 mT northward at 60.08 below the horizontal in Atlanta, Georgia. A tube in a neon sign stretches between two diagonally opposite corners of a shop window—which lies in a north-south vertical...
  45. S

    How Do You Calculate the Speed of a Rod in a Magnetic Field?

    Homework Statement A rod of mass 0.720 kg and radius 6.00 cm rests on two parallel rails that are d=12.0 cm apart and L=45.0 cm long. The rod carries a current of I=48.0 A in a direction that makes it rolls along the rails without slipping. A uniform magnetic field of magnitude 0.240 T is...
  46. S

    What is the Correct Way to Calculate Magnetic Force on an Electron or Proton?

    <Moderator's note: Moved from a technical forum and thus no template.> So there was this question An electron that has velocity (2*10^6 i +3*10^6 j)m/s moves through the uniform magnetic field (0.03 i - 0.15 j) T (a) Find the force on the electron due to the magnetic field. (b) Repeat your...
  47. E

    Direction of Magnetic Force on a Wire Moving Down a Slope with Current I?

    Homework Statement A wire with a current of I (flowing into the screen) moves down on a slope. What is the direction of the magnetic force acting on it? Homework EquationsThe Attempt at a Solution I have no Idea why the answer isn't horizontal, but rather opposing its velocity direction.
  48. H

    Find induced current, magnetic force, work in inclined plane

    Homework Statement [IMG]http://[url=https://ibb.co/b3Emfo]https://preview.ibb.co/gLaEY8/20180527_055248.jpg[ A conducting bar slides down without friction on a pair of conducting rails separated by distance d. Connected to resistor R and there magnetic field B directed upward, perpendicular to...
  49. P

    Current Carrying Wires: How Ampere's Law Affects Safety

    Why don't wires in any circuit touch each other and get short circuited? If two current carrying wires carry charges in same direction then from ampere's law,can't they touch each other due to attractive forces?
Back
Top